Easy Limit Question (Final Value Theorem)

In summary, the conversation discusses the use of the Final Value Theorem to find the limit of a function. However, it is determined that the FVT cannot be applied in this case due to the function's divergence as s approaches 0. The speaker had hoped to use the FVT to confirm their previous answer, but it is concluded that this is not possible.
  • #1
Saladsamurai
3,020
7

Homework Statement



I am given finction in the Laplace domain

[tex]X(s) = \frac{3s+7}{s^2(s+9)}[/tex]

and I am asked to find:

[tex]\lim_{t\rightarrow\infty}x(t)[/tex]

I solved this by partial fraction expansion and transformed it to the time domain, took the limit and the result was an infinite limit.

I feel like I could have used the Final Value Theorem which says that [itex]\lim_{t\rightarrow\infty}x(t) = \lim_{s\rightarrow 0}X(s)[/itex] and made this easier. Does anyone see how? As it stands, I cannot evaluate the limit as s-->0 because of the denominator. But if I could get it into an 'indeterminate form' I could use L'Hopital's rule.

Any thoughts?
 
Last edited:
Physics news on Phys.org
  • #2
I think you have a typo in the function. In any case, what's the problem? Both methods gave you the same result: x(t) diverges as [itex]t\rightarrow\infty[/itex].
 
  • #3
Hi vela. I don't you read what I wrote correctly. Perhaps take another look. :wink:

[typo fixed]
...what's the problem? Both methods gave you the same result

The whole point of the thread is that I did not use FVT because I do not know how to get X(s) into a form that allows me too. So how would I know that both methods yield the same result?

I appreciate your help :smile:

Casey
 
  • #4
You can't use the final value theorem because the limit of sX(s) doesn't exist as s approaches 0. The top approaches 7 while the bottom goes to 0. There's no way to get around it diverging.

I'm not sure why you're expecting to be able to produce a finite answer via the FVT. You already found that x(t) diverges as t goes to infinity by explicitly finding x(t) and taking its limit.
 
  • #5
vela said:
You can't use the final value theorem because the limit of sX(s) doesn't exist as s approaches 0. The top approaches 7 while the bottom goes to 0. There's no way to get around it diverging.

I'm not sure why you're expecting to be able to produce a finite answer via the FVT. You already found that x(t) diverges as t goes to infinity by explicitly finding x(t) and taking its limit.

I am not expecting to produce a finite answer. I thought I might be able to confirm my answer using FVT. But I think that I agree that there isn't anyway to re-write X(s) such that L'Hopital's rule could be used.

Do you know if FVT only works for finite-valued limits?

Thanks again :smile:
 
  • #6
Saladsamurai said:
I am not expecting to produce a finite answer. I thought I might be able to confirm my answer using FVT. But I think that I agree that there isn't anyway to re-write X(s) such that L'Hopital's rule could be used.
Well, in a sense you did. Applying the FVT says the limit diverges, which is consistent with what you found by taking the limit of x(t).

Do you know if FVT only works for finite-valued limits?
To say that the limits are equal assumes that the limits exist, but I'd guess that if one limit blows up, the other one will too. I've seen various statements of the theorem. Most seem to say it applies if the poles lie in the LHS of the complex plane, which isn't true for your particular X(s). The other statements are sloppier, just stating the result without the necessary givens.
 

1. What is the final value theorem and how does it relate to easy limit questions?

The final value theorem is a mathematical tool used to determine the steady-state behavior of a system. In easy limit questions, the final value theorem can be used to find the value of a function as it approaches infinity.

2. How do I know when to apply the final value theorem in an easy limit question?

The final value theorem can be applied when the limit question involves a rational function, and the limit is being taken as x approaches infinity.

3. Can the final value theorem be used for more complex functions?

No, the final value theorem is only applicable for rational functions, which are functions that can be expressed as a ratio of two polynomials.

4. Is the final value theorem the only way to solve easy limit questions?

No, there are other methods such as L'Hopital's rule and graphing that can also be used to solve easy limit questions.

5. Are there any limitations or assumptions when using the final value theorem for easy limit questions?

Yes, the final value theorem assumes that the system is stable and that the limit exists. It also does not work for functions with non-zero initial conditions.

Similar threads

  • Calculus and Beyond Homework Help
Replies
8
Views
665
  • Calculus and Beyond Homework Help
Replies
20
Views
1K
  • Calculus and Beyond Homework Help
Replies
4
Views
966
  • Calculus and Beyond Homework Help
Replies
12
Views
785
  • Calculus and Beyond Homework Help
Replies
16
Views
1K
  • Calculus and Beyond Homework Help
Replies
2
Views
160
  • Calculus and Beyond Homework Help
Replies
26
Views
2K
  • Calculus and Beyond Homework Help
Replies
11
Views
1K
  • Calculus and Beyond Homework Help
Replies
14
Views
1K
  • Calculus and Beyond Homework Help
Replies
8
Views
802
Back
Top